24 votos

Dejando $S(m)$ ser la suma de dígitos de $m$,, a continuación,$\lim_{n\to\infty}S(3^n)=\infty$?

Para cualquier $m\in\mathbb N$, vamos a $S(m)$ ser la suma de dígitos de $m$ en el sistema decimal.

Por ejemplo, $S(1234)=1+2+3+4=10, S(2^5)=S(32)=5$.

Pregunta 1 :Es el verdadero? $$\lim_{n\to\infty}S(3^n)=\infty.$$

Pregunta 2 : ¿Cómo sobre la $S(m^n)$ para$m\ge 4$, salvo en algunos casos triviales?

Comentario : Esta pregunta se ha hecho anteriormente en las matemáticas.SE sin recibir respuestas completas, donde spin demostrado que $\lim_{n\to\infty} \sup S(m^n )=\infty$ al $m$ no es una potencia de $10$.

https://math.stackexchange.com/questions/501019/letting-sm-be-the-digit-sum-of-m-then-lim-n-to-inftys3n-infty

Motivación : tengo las siguientes : $$\lim_{n\to\infty}S(2^n)=\infty.$$

Prueba : El objetivo de esta prueba es que no existe un número distinto de cero entre el ${m+1}^{th}$ dígitos y ${4m}^{th}$ dígitos.

Si $$2^n=A\cdot{10}^{4m}+B, B\lt {10}^m, 0\lt A,$$ a continuación, $2^n\ge {10}^{4m}\gt 2^{4m}$ lleva $n\gt 4m$. Por lo tanto, el lado izquierdo se puede dividir por $2^{4m}$. También, $B$ debe ser dividido por $2^{4m}$ porque ${10}^{4m}=2^{4m}\cdot 5^{4m}$. Sin embargo, desde $$B\lt {10}^m\lt {16}^m=2^{4m},$$ $B$ no puede ser dividido por $2^{4m}$ si $B\not=0$. Si $B=0$, entonces el lado derecho se puede dividir por $5$, pero la izquierda no puede ser dividido por $5$. Por lo tanto, ahora sabemos que no es un número distinto de cero entre el ${m+1}^{th}$ dígitos y ${4m}^{th}$ dígitos. Desde $2^n$ no puede ser dividido por $5$, el primer dígito es el no $0$. No existe número distinto de cero entre el segundo dígito y el cuarto dígito. Una vez más, no existe número distinto de cero entre el $5^{th}$ dígitos y ${16}^{th}$ dígitos. Por el mismo argumento anterior, si $2^n$ tiene más de $4^k$ dígitos, a continuación,$S(n)\ge {k+1}$. Por lo tanto, $$n\log {2}\ge 4^k-1\ \ \Rightarrow \ \ S(n)\ge k+1.$$ Ahora sabemos que $$\lim_{n\to\infty}S(2^n)=\infty$$ como se desee. Ahora la prueba se ha completado.

Sin embargo, he estado en la dificultad para la $m=3$ de los casos. Tengo $\lim\sup S(3^n)=\infty$.

Prueba : Supongamos que $3^n$ ha $m$ dígitos. Dejando $l=\varphi({10}^m)+n$, luego $$3^l-3^n=3^n(3^{\varphi({10}^m)}-1).$$ Ya que este puede ser dividido por ${10}^m$, sabemos que la última $m$ dígitos de $3^l$ son iguales a los de $3^n$. Por lo tanto, obtenemos $\lim\sup S(3^n)=\infty$.

Sin embargo, yo no puedo ir $\lim_{n\to\infty}\inf S(3^n)$. Alguien puede ayudar?

38voto

myhd Puntos 1948

Esto se deduce de la W. M. Schmidt Subespacio teorema, que es una profunda teorema de diophantine aproximaciones generalización de Roth a diversas variables. Un relato completo de este teorema y su prueba, así como algunas de sus sorprendentes aplicaciones, se puede encontrar en el capítulo 7 de Alturas en Diophantine Geometría por Bombieri y Gubler. El siguiente resultado, la finitud de la cantidad de no-degenerada soluciones a los llamados "$S$-unidad de la ecuación," es una aplicación directa de Schmidt del teorema. (Ver Teorema 7.4.2 en [HIDG]):

Deje $S$ ser un conjunto finito de números primos, y corregir $n \in \mathbb{N}$. Considere la posibilidad de $\mathcal{X}$ el conjunto de soluciones a $x_1 + \cdots + x_n = 1$ en los números racionales $x_i$ de la forma $\pm \prod_{p \in S} p^{a_p}$, $a_p \in \mathbb{Z}$, de tal manera que ningún adecuada subsum de $x_1+\cdots+x_n$ se desvanece. A continuación, $\mathcal{X}$ es un conjunto finito.

Esto implica a su pregunta de inmediato al considerar $S := \{2,3,5,7\}$.

Sin embargo, la prueba de la Subespacio teorema no es eficaz, y esto sólo muestra $S(3^n) \to +\infty$ sin ningún tipo de estimación a la baja en la tasa de crecimiento. Un límite inferior en $S(3^n)$ (va hacia el infinito con $n$) está disponible a través de Baker método; es debido a Stewart, y la búsqueda de google me llevó a la antigua MathOverflow post enlazado en mi comentario de abajo. Me limitaré a copiar las referencias que Gerry Myerson suministrados:

C. L. Stewart, En la representación de un número entero en dos bases diferentes, J Reine Angew Matemáticas 319 (1980) 63-72, SEÑOR 81j:10012; H G Senge, E G Straus, PV-números y conjuntos de la multiplicidad, los Procedimientos de la Universidad del Estado de Washington Conferencia sobre la Teoría de los números (1971) 55-67, MR 47 #8452.

3voto

Alexey Ustinov Puntos 3951

No es más sencillo de lo agrument que resuelve el problema: potencias $m^n$ pueden empezar con una cadena arbitraria de dígitos.

0voto

joschi Puntos 878

Sólo quiero dar otra prueba a su segunda pregunta. Deje $SS(m^n)=\lbrace S(m^n) ; n\geq1\rbrace$. Nos muestran que este conjunto es ilimitado. Primero un Lema:

$Lemma:$ Deje $a$ ser cualquier entero arbitrario, a menos que sus casos triviales. Para cualquier entero arbitrario $b$, no es un número entero $t$ tal que $a^t$ comienza con $b$.

Ahora, por el contrario, supongamos que $Max$ $SS(m^{n_1})=M$ para algunos $n_1\in N$, y ver sólo esta secuencia de enteros:

$\overline {m^{n_1}1}, \overline {m^{n_1}11}, \overline {m^{n_1}111}, \ldots$

esta secuencia es infinito y por el lema anterior, para cada uno de ellos debe tener un adecuado $t_i$, $i$ es la posición de cada número en la secuencia de arriba, que $m^{t_i}$ comienza con $i$-ésimo término en las secuencias. Obviamente, cada uno de ellos tiene el dígito de la suma mayor que $M$ y es una contradicción. Esto completa la prueba.

i-Ciencias.com

I-Ciencias es una comunidad de estudiantes y amantes de la ciencia en la que puedes resolver tus problemas y dudas.
Puedes consultar las preguntas de otros usuarios, hacer tus propias preguntas o resolver las de los demás.

Powered by:

X